Buscar

Eletricidade - Superior

Prévia do material em texto

Questão 1: Um disco de raio a está carregado uniformemente com 
uma carga Q. Calcule o vetor campo elétrico num ponto P sobre o 
eixo de simetria perpendicular ao plano do disco a uma distância z 
do seu centro. Nesse mesmo ponto calcule o potencial elétrico. 
 
 
Resposta: 
Com base na figura, podemos obter as seguintes informações a serem utilizadas 
no problema: 
 
• Carga do disco: Q; 
• Raio do disco: A; 
• Distância do disco ao ponto P (onde se quer o potencial elétrico): Z 
 
Como informado na questão, o disco está carregado uniformemente com uma 
carga Q. Dessa maneira, podemos chamar de dq um elemento de carga 
presente no disco. 
 
Com base na figura podemos perceber agora que, “a” (raio do disco) representa 
a distância da origem ao elemento de carga dq. 
 
Podemos também analisar a distância do elemento de carga dq ao ponto P 
(onde se quer o potencial elétrico): 
 
Podemos chamar essa distância (linha verde) de R. 
 
Observando a figura, fica claro que podemos aplicar o Teorema de Pitágoras ao 
triângulo que montamos. Dessa maneira teremos que: 
𝑎2 + 𝑧2 = 𝑅2 
(𝑎2 + 𝑧2)
1
2 = 𝑅 
 
Sabemos que o Potencial elétrico é dado por: 
𝑉 = 
1
4𝜋𝜖0
∫
𝑑𝑞
𝑟
 
 
Para obtermos o elemento de carga dq, podemos recorrer a expressão da 
densidade linear de carga. Nesta expressão temos um elemento de arco de 
ângulo dθ do disco. Esse elemento de arco pode ser observado na figura abaixo: 
 
Logo, temos que a expressão da densidade linear de carga pode ser escrita da 
seguinte maneira: 
λ = 
𝑑𝑞
𝑑𝑠
 
Então, temos que: 
dq = λds 
E, com base na figura e na explicação anterior, que: 
dq = λa dθ 
Após as substituições necessárias na expressão do potencial elétrico, temos 
que: 
𝑉 = 
1
4𝜋𝜖0
∫
λa
(𝑎2 + 𝑧2)
1
2
dθ 
Organizando a integral acima, vemos que a densidade de carga e o raio são 
constantes. Já os limites de integração corresponderão a uma volta completa no 
disco (0 a 2π). Dessa maneira temos que: 
𝑉 = 
1
4𝜋𝜖0
λa
(𝑎2 + 𝑧2)
1
2
∫ dθ
2π
0
 
Resolvendo a integral, temos que: 
∫ dθ
2π
0
= 2π − 0 = 2π 
𝑉 = 
1
4𝜋𝜖0
λa
(𝑎2 + 𝑧2)
1
2
 2π 
𝑉 = 
1
4𝜋𝜖0
2πλa
(𝑎2 + 𝑧2)
1
2
 
Sabemos que a carga do disco é Q e que tem comprimento igual a 2πa. Dessa 
maneira, temos que a densidade linear de carga (λ) é: 
λ = 
𝑄
2πa
 
 
 
Logo: 
 𝑄= 2πaλ 
 
Substituindo na integral, temos que o potencial elétrico no ponto P é: 
𝑽 = 
𝟏
𝟒𝝅𝝐𝟎
𝑸
(𝒂𝟐 + 𝒛𝟐)
𝟏
𝟐
 
 
Seguindo a questão, agora encontraremos o vetor campo elétrico no ponto P. 
Para isso, utilizaremos o gradiente de potencial, que é dado por: 
𝑬 = −∇𝑉 
∇ (nabla) é dado por: 
(
𝜕
𝜕𝑥
𝑖 +
𝜕
𝜕𝑦
𝑗 +
𝜕
𝜕𝑧
𝑘) 
Logo, o gradiente de potencial é dado por: 
𝑬 = −(
𝜕
𝜕𝑥
𝑖 +
𝜕
𝜕𝑦
𝑗 +
𝜕
𝜕𝑧
𝑘)𝑉 
𝑬 = −(
𝜕𝑉
𝜕𝑥
𝑖 +
𝜕𝑉
𝜕𝑦
𝑗 +
𝜕𝑉
𝜕𝑧
𝑘) 
A derivada parcial com relação a x é: 
𝜕𝑉
𝜕𝑥
= 
𝜕
𝜕𝑥
 
1
4𝜋𝜖0
𝑄
(𝑎2 + 𝑧2)
1
2
 
 
A carga (Q), a permissividade do meio (𝝐𝟎), o raio do disco (a) e a distância (z) 
são constantes (a derivada é em relação a x). Sabemos que a derivada de uma 
constante é 0, logo: 
𝜕𝑉
𝜕𝑥
= 0 
 
Podemos observar que para 
𝜕𝑉
𝜕𝑦
 cairemos na mesma situação de 
𝜕𝑉
𝜕𝑥
. 
Para 
𝜕𝑉
𝜕𝑧
 temos que: 
𝜕𝑉
𝜕𝑧
= 
𝜕
𝜕𝑧
 
1
4𝜋𝜖0
𝑄
(𝑎2 + 𝑧2)
1
2
 
A carga (Q) e a permissividade do meio (𝝐𝟎 são constantes, então podemos 
retira-las da derivada: 
𝜕𝑉
𝜕𝑧
=
𝑄
4𝜋𝜖0
 
𝜕
𝜕𝑧
 
1
(𝑎2 + 𝑧2)
1
2
 
A função V(z), pela regra da cadeia, é: 
𝑑𝑉[𝑤(𝑧)]
𝑑𝑧
= 
𝑑𝑉
𝑑𝑤
 
𝑑𝑤
𝑑𝑧
 
• V(w) = 
1
𝑤
1
2
 
• w(z) = a²+z² 
Seguindo, as derivadas ficarão da seguinte maneira: 
𝑑𝑉
𝑑𝑤
=
𝑑(𝑤−
1
2)
𝑑𝑤
= −
1
2
𝑤−
1
2
−1 = −
1
2
𝑤
−1−2
2 = −
1
2
𝑤−
3
2 = −
1
2
1
𝑤
3
2
 
𝑑𝑤
𝑑𝑧
= 2𝑧 
𝜕𝑉
𝜕𝑧
=
𝑄
4𝜋𝜖0
[−
1
2
 
1
(𝑎2 + 𝑧2)−
3
2
2𝑧] = 
1
4𝜋𝜖0
 
𝑄𝑧
(𝑎2 + 𝑧2)−
3
2
 
Logo, o vetor campo elétrico no ponto P é dado por: 
𝑬 = − (−
1
4𝜋𝜖0
 
𝑄𝑧
(𝑎2 + 𝑧2)
1
2
𝑘) 
 
𝑬 = 
𝟏
𝟒𝝅𝝐𝟎
 
𝑸𝒛
(𝒂𝟐 + 𝒛𝟐)
𝟏
𝟐
𝒌 
Questão 2: Determine o módulo do campo elétrico em todo o espaço 
gerado por uma esfera maciça carregada com uma carga Q 
distribuída uniformemente pelo seu volume. 
Resposta: 
Pelo enunciado, temos que: 
• Carga da esfera: Q 
A lei de Gauss nos mostra que: 
∮ 𝐸. 𝑑. 𝐴 = 
𝑞
𝜖0𝐴
 
Admitindo que a carga Q seja positiva e que o raio da esfera seja R, 
precisaremos levar em consideração os pontos exteriores a esfera (pontos onde 
r > R) que será uma superfície Gaussiana exterior e os pontos no interior da 
mesma (onde r ≤ R) que será uma superfície Gaussiana interna para que assim 
possamos determinar o módulo do campo elétrico como a questão pede. 
 
Pela lei de Gauss, apenas a carga interna da superfície pode contribuir com o 
campo elétrico. Sendo ρ a densidade volumétrica de cargas, podemos 
reescrever a integral com o volume limitado por essa superfície Gaussiana da 
seguinte forma: 
∮ 𝐸. 𝑑. 𝐴 = 
1
𝜖0
∫ ρ dV
𝐴
 
Sabemos que a superfície Gaussiana que passa pelo ponto onde queremos 
calcular o campo tem um raio, que chamaremos de rs. Por outro lado, 
chamaremos de rq o raio da superfície Gaussiana atrelada a distribuição interna 
das cargas. Como mostra a figura abaixo, esses raios irão coincidir, sendo rs = 
rq: 
 
 
Imagine que a figura acima desenhada é uma esfera e que as setas em marrom 
são er. Essas setas apontam para fora da esfera. O campo elétrico vai se 
espalhar de maneira radial, a partir da distribuição das cargas, na direção er. Na 
superfície da esfera também temos elementos da área que chamaremos de dA, 
cada elemento de área possui um vetor unitário n (indicado pela seta vermelha) 
que é perpendicular a superfície e orientado para fora da esfera. O vetor campo 
elétrico terá e mesma direção e sentido desse vetor unitário n. 
 
O vetor elemento de área é: 
𝑑𝑨 = 𝑑𝐴𝑛 
A componente do vetor campo elétrico, na direção de er, será: 
𝑬 = 𝐸𝐞𝐫 
Fazendo as devidas substituições em: 
∮ 𝐸. 𝑑. 𝐴 = 
1
𝜖0
∫ ρ dV
𝐴
 
Temos que: 
∮ 𝐸 𝑒𝑟. 𝑑𝐴 𝑛 = 
1
𝜖0
∫ ρ dV
𝐴
 
 
Como dito mais acima além de n e er serem vetores unitários, ambos estão na 
mesma direção e sentido (ângulo entre eles é igual a 0). 
Logo, er .n = ∣er∣∣n∣ cos0 = 1 
 
Reescrevendo, temos que: 
∮ 𝐸 𝑑𝐴 = 
1
𝜖0
∫ ρ dV
𝐴
 
 
Estudando o elemento dV (distribuição de cargas internas a superfície 
Gaussiana) temos que: 
𝑑𝑉 = 𝑟q 𝑑𝜃 × 𝑟q𝑠𝑒𝑛𝜃 𝑑𝜑 𝑑𝑟q 
𝑑𝑉 = 𝑟2q 𝑠𝑒𝑛𝜃 𝑑𝜑 𝑑𝜃 𝑑𝑟q 
 
Estudando o elemento dA (superfície Gaussiana) temos que: 
 
𝑑𝐴 = 𝑟s 𝑑𝜃 × 𝑟s𝑠𝑒𝑛𝜃 𝑑𝜑 
𝑑𝐴 = 𝑟2s 𝑠𝑒𝑛𝜃 𝑑𝜑 𝑑𝜃 
Substituindo em: 
∮ 𝐸 𝑑𝐴 = 
1
𝜖0
∫ ρ dV
𝐴
 
Temos que: 
∫ E 𝑟2s 𝑠𝑒𝑛𝜃 𝑑𝜑 𝑑𝜃 =
1
𝜖0
∫ ρ 𝑟q2 𝑠𝑒𝑛𝜃 𝑑𝜑 𝑑𝜃 𝑑𝑟q 
 
A integral não depende do raio da superfície Gaussiana antes da igualdade, já 
após a igualdade, ela não depende da densidade volumétrica de cargas. 
Também não existem termos cruzados em 𝜃 𝑒 𝜑 nos dois lados e em 𝑟q após 
a igualdade (podendo separar as integrais). Dessa maneira, podemos reescrever 
como: 
 
E 𝑟2s ∫ 𝑠𝑒𝑛𝜃 𝑑𝜃 ∫ 𝑑𝜑 =
ρ
𝜖0
∫ 𝑟q2 𝑑𝑟q ∫ 𝑠𝑒𝑛𝜃 𝑑𝜃 ∫ 𝑑𝜑 
 
 
Como estamos tratando de uma esfera, os limites de integração serão de 0 a 
𝜋 em 𝑑𝜃 e de 0 a 2𝜋 (representando uma volta completa) em 𝑑𝜑 (ambos do 
lado esquerdo da igualdade). 
 
Já do lado direito da igualdade, teremos os limites de 0 a 𝜋 em 𝑑𝜃 e de 0 a 2𝜋 
em 𝑑𝜑 (também representando uma volta completa). Em 𝑑𝑟q teremos limites 
de 0 a r. 
 
Reescrevendo, temos que: 
E 𝑟2s ∫ 𝑠𝑒𝑛𝜃 𝑑𝜃 ∫ 𝑑𝜑 =
ρ
𝜖0
2𝜋
0
𝜋
0
 ∫ 𝑟q2 𝑑𝑟q ∫ 𝑠𝑒𝑛𝜃 𝑑𝜃
𝜋
0
𝑟
0
∫ 𝑑𝜑
2𝜋
0
 
Para ∫ 𝑠𝑒𝑛𝜃 𝑑𝜃𝜋
0
 temos que: - (cosπ – cos 0) = 2 
Para ∫ 𝑑𝜑2𝜋
0
 temos que: 2π-0 = 2π 
Para ∫ 𝑟2q 𝑑𝑟q𝑟
0
 temos que: 
𝑟q2+12 + 1
 =
𝑟³
3
 
Reorganizando, temos que: 
E 𝑟22.2𝜋 =
ρ
𝜖0
𝑟³
3
2.2𝜋 
E 𝑟24𝜋 =
ρ
𝜖0
𝑟³
3
4𝜋 
E =
ρ
𝜖0
𝑟
3
 
Sabendo que a densidade volumétrica de cargas é ρ =
Q
𝑉
 , a totalidade de carga 
é distribuída em uma esfera que possui volume igual a V =
4
3
𝜋𝑅³. Dessa 
maneira, podemos escrever a densidade de cargas em função do raio de 
distribuição e da carga total como: ρ =
Q
4
3
𝜋𝑅³
 ou ρ =
3Q
4𝜋𝑅³
 . 
Substituindo, temos que para r ≤ R: 
E =
1
𝜖0
3Q
4𝜋𝑅³
𝑟
3
 
𝐄 =
𝐐
𝟒𝝅𝝐𝟎𝑹³
𝒓 
Para r > R, podemos utilizar: 
 
E 𝑟2s ∫ 𝑠𝑒𝑛𝜃 𝑑𝜃 ∫ 𝑑𝜑 =
ρ
𝜖0
∫ 𝑟q2 𝑑𝑟q ∫ 𝑠𝑒𝑛𝜃 𝑑𝜃 ∫ 𝑑𝜑 
 
 
A superfície Gaussiana passa pelo ponto onde queremos calcular o campo 
elétrico, tendo raio rs = r externo a distribuição das cargas. O raio de distribuição 
de cargas é o raio da esfera rq = R. 
 
Calculando a integral com os limites de integração teremos: 
E 𝑟2s ∫ 𝑠𝑒𝑛𝜃 𝑑𝜃 ∫ 𝑑𝜑 =
ρ
𝜖0
2𝜋
0
𝜋
0
 ∫ 𝑟q2 𝑑𝑟q ∫ 𝑠𝑒𝑛𝜃 𝑑𝜃
𝜋
0
𝑅
0
∫ 𝑑𝜑
2𝜋
0
 
 
Para ∫ 𝑟q2 𝑑𝑟q𝑅
0
 temos que: 
𝑟q2+1
2 + 1
 =
𝑅³
3
 
 
Reorganizando, temos que: 
E 𝑟22.2𝜋 =
ρ
𝜖0
𝑅³
3
2.2𝜋 
E 𝑟24𝜋 =
ρ
𝜖0
𝑅³
3
4𝜋 
E =
ρ
𝜖0
𝑅
3
 
 
Substituindo, temos que para r > R: 
E =
1
𝜖0
3Q
4𝜋𝑅³
𝑅
3
 
𝐄 =
𝐐
𝟒𝝅𝝐𝟎𝒓²
 
 
 
 
 
Questão 3: Um fio com área de seção transversal 0,5 × 10−2 cm², é 
percorrido por uma corrente contínua de intensidade igual a 4,0 A. 
Dada a carga elementar 1,6 × 10−19C, determinar: 
 
a) O número de elétrons passando por uma seção transversal por segundo: 
 
Resposta: 
 
Com base no enunciado temos que: 
• Área transversal do fio (A): 0,5 × 10−2 
• Corrente (i): 4ª 
• Carga elementar (e): 1,6 × 10−19 
 
A corrente elétrica pode ser encontrada por 𝑖 =
∆𝑞
∆𝑡
; 
A quantidade de cargas atravessando uma seção: ∆𝑞 = 𝑛𝑒 
 
Substituindo, temos que: 𝑖 =
𝑛𝑒
∆𝑡
; 𝑛 =
𝑖∆𝑡
𝑒
; 
Determinando ∆𝑡= 1s, temos que: 
 
𝒏 =
𝟒 × 𝟏
𝟏, 𝟔 × 𝟏𝟎−𝟏𝟗
= 𝟐, 𝟓 × 𝟏𝟎𝟏𝟗 
 
 
 
 
 
 
 
 
b) A velocidade média dos elétrons, sabendo que existem 𝟏, 𝟖 ×
𝟏𝟎𝟐𝟎elétrons/cm³: 
Resposta: 
Sabemos que a velocidade média pode ser encontrada por: 
𝑣𝑚 =
∆𝑠
∆𝑡
 
O problema nos informa que o elétron percorre uma seção transversal, dessa 
maneira ele vai percorrer um ∆𝑠 em um ∆𝑡. A seção transversal vai formar um 
cilindro no fio com volume V. Dessa maneira, temos que: 
𝑉 = 𝐵ℎ 
A densidade volumétrica de cargas é dada por: 
𝑑 =
𝑛
𝑉
 
Sendo h o deslocamento dos elétrons (∆𝑠) e B a área da base do cilindro (seção 
transversal). Teremos então que: 
∆𝑠 =
𝑉
𝐴
 
Substituindo: 
𝑣𝑚 =
𝑉
𝐴∆𝑡
 
 
Substituindo novamente, temos: 
𝑣𝑚 =
𝑛
𝑑𝐴∆𝑡
 
Determinando ∆𝑡= 1s, temos que: 
𝑣𝑚 =
2,5 × 1019
1,8 × 1020. 0,5 × 10−2. 1
 
𝑣𝑚 = 0,28 × 102 
 
𝒗𝒎 = 𝟐𝟖𝒄𝒎/ 
Questão 4: No circuito abaixo determinar as correntes nos ramos e seus 
verdadeiros sentidos: 
 
Resposta: 
Com base no circuito apresentado temos que: 
Resistores: R1 = 0,5 Ω; R2 = 0,5 Ω; R3 = 1 Ω; R4 = 0,5 Ω; R5 = 0,5 Ω; R6 = 3 Ω; 
R7 = 1 Ω. 
 
Fontes de Tensão: E1 = 20 V; E2 = 20 V; E3 = 6 V; 
 
Determinando aleatoriamente o sentido das correntes: 
 
 
Dessa maneira, podemos dizer que i1 + i3 = i2. 
 
Analisando apenas a malha A, no sentido horário, temos que: 
𝑅2𝑖1 + 𝑅4𝑖2 + 𝐸2 + 𝑅5𝑖2 + 𝑅3𝑖1 + 𝑅1𝑖1 − 𝐸1 = 0 
0,5𝑖1 + 0,5𝑖2 + 20 + 0,5𝑖2 + 1𝑖1 + 0,5𝑖1 − 20 = 0 
2𝑖1 + 𝑖2 = 0 
 
Analisando apenas a malha B, no sentido horário, temos que: 
−𝑅6𝑖3 + 𝐸3 − 𝑅7𝑖3 − 𝑅5𝑖2 − 𝐸2 − 𝑅4𝑖2 = 0 
−3𝑖3 + 6 − 1𝑖3 − 0,5𝑖2 − 20 − 0,5𝑖2 = 0 
−𝑖2 − 4𝑖3 = 14 
Dessa maneira, chegamos ao sistema: 
{
𝑖1 + 𝑖3 = 𝑖2
2𝑖1 + 𝑖2 = 0
−𝑖2 − 4𝑖3 = 14
 
Isolando i2 na 3ª equação: 
𝑖3 = 
−14 − 𝑖2
4
 
Isolando i1 na 2ª equação temos que: 
𝑖1 = 
−𝑖2
2
 
Substituindo na equação 1: 
𝑖2 = −
𝑖2
2
+
(−14 − 𝑖2)
4
 
 
−𝑖2 −
𝑖2
2
+
(−14 − 𝑖2)
4
= 0 
Resolvendo o MMC: 
−4𝑖2 − 2𝑖2 − 14 − 𝑖2
4
= 0 
−4𝑖2 − 2𝑖2 − 14 − 𝑖2
4
= 0 
−4𝑖2 − 2𝑖2 − 14 − 𝑖2 = 0 × 4 
−7𝑖2 − 14 = 0 
𝒊𝟐 = −𝟐𝑨 
Substituindo: 
𝑖1 = −
(−2)
2
 
𝒊𝟏 = 𝟏𝑨 
Substituindo: 
𝑖3 =
−14 − (−2)
4
 
𝒊𝟑 = −𝟑𝑨 
Sendo os valores de i2 e i3 negativos, podemos perceber que o sentido real dos 
mesmos é o contrário ao adotado para resolver o circuito, sendo os seus valores 
reais: 
𝒊𝟏 = 𝟏𝑨 
𝒊𝟐 = 𝟐𝑨 
𝒊𝟑 = 𝟑𝑨 
O sentido das correntes no circuito é:

Continue navegando